You are on page 1of 8
Date : 13/06/2022 ® Aakash +hsyus Corporate Office: Aakash Tower, 8, Pusa Road, New Delhi-110005 Ph. 011-47623456 Olympiad-Classroom Assessment Practice Sheet O-CAPS-03 : PRMO / |IOQM (For Vill, IX & X Studying Students) Topics Covered Mathematics : Sets, Number Theory MATHEMATICS. 1. How many non-empty subsets ofthe sel A = (1, 2)3, 4, 5, 6) has atmost’4 elements in it? 2. Let Pe a prime number such that the number P24 11 has\exaetly 6 positive divisors. Find P 3. Consider the set A= {1, 4, 7, 10, 13, 18 .97, 100}. All subsets of A having IV elements have the property that there are two elements in the set whose sum is 104. Find the minimum value of N. 4. Let p be a prime number such that p ~ 1 has exactly 10 positive factors while p + 1 has exactly 6 positive pit factors. Find the value of 2016 5. Find the least prime factor of the number 2078 = 3 6. For how many values of mis 12"? the least common multiple of the positive integers 6°, 8° and m? 7. Let the Set $= (5, 8, 1, 13, 34, 21, 3, 2}, Mira makes a list as follows: For each two element subset of S, she writes on her list the greater of the set’s two elements. If the sum of the numbers on the list is k, then find the sum of digits of k. 8 Let Set P be a 90-element subset of {1, 2, 3, .......100} and let S be the sum of the elements of P. If the number of possible value of Sis m, then find the sum of digits 9. How many positive perfect squares less than 10° are multiples of 247 10. If n(A= B) = 18, n(A vB) = 80 and n(AnB) = 30, then find n(B). Pa! ae " 12. Let p and q be two positive integers. For some fixed integer n, the sel {n ~ 1, 3n ~ 19, 38 - 5n, 7n ~ 45} represents {p, 2p, 4, 29} but not necessarily in that order. Find the value of n. (For Vill, IX & X Studying Students) O-CAPS.03 : PRMO / OOM 13. Alica has 6 real numbers a 3, we know that P is either 6k + 1 or Case: 6k —1 form Ifp—1 is a multiple of 3 then p 4 x 31 or 2 So, N= P+ 11= 36 (12k+1 +11) * 3! Le,, p + 1 = 50 or 164 of which both have multiple of 12 exactly 6 positive divisors but p + 1 = 50 gives = 129 (say) p= 49 whichis not prime so, p = 163 Then it has more than 6 factors Cases: So, P= Sis the only possibilty If p + 1 is a multiple of 3, then p + 1 = 2 x 3! 3. Answer (19) The elements of set A may be partitioned in this manner. {1), {52}, {4, 100), {7, 97}, (10, 94), ...... 449, 55} So, or 2" x 3i.e., p~1= 10 or 16 Neither of which has exactly 10 positive divisors, = 163 Is the only possibilty (O-CAPS.03 : PRMO / IOQM - Ans & Sol (For Vil, 1X & X Studying Students) 3, ‘Answer (11) 20162018 3 Let, N= 372% 2016715 — 4 Also, 2016 = 2° x 3? x 7! So, Nis not a multiple of 2, 3, 7 Also, the last digit of Nis 1 and hence not a multiple of 5 Now, 2016%'° = 32°15 (mod 11) = (35) (mod 11) = 243*°5(mod 11) = 1 (mod 11) So, N= (672 «1 —1) (mod 11) = 671 (mod 11) 0 (mod 11) Le., Nis a multiple of 11 Answer (25) mhas only 2s and 3s in its prime factorization, or m= 29° 6-28.38 ah = 2% 422 = 224 gt? The LCM of any numbers can be fotind by writing out their factorizations and taking the greatest ower for each factor (65,8) =2% 3° 12°? = 224 2 (92498, 9° 2b] pmanl24, alg maxis, ) 2 Since 0 mM(B-A)=32 n{B) = (ANB) + n(B- A) ‘Answer (07) Since all elements of {p, 4, 2p, 2q) are positive, so 38 -5n > O and 7n- 45> 0 ie, Ben <8 " 5 But n being an integer, son =7 Also, putting n = 7, the set becomes {6, 2, 3, 4} which satisfies the given conditions. (For Vill, IX & X Studying Students) (O-CAPS.03 : PRMO / IOQM - Ans & Sol 73. 14. 16. Answer (65) 76 The largest two are d+ eand e+e So,d+e=51andc+e=48 The smallest two are a+ b< ate So,a+b=32anda+e=36 The third smallest is either a+ dor b+¢ Now, (d+ €)+(a+e)=(c+e)+(a+d) ie, 1 +36 =48 +(a+d) 1. ie, a+d=39 So,b+0=37 Now, 2e = (d+ e) + (c+ e)—(c +d) = 51+ 48-[(a+d)+ (b+ )—(a+ B)) = 51+ 48 - [39 + 37-32] =55 Answer (76) First we consider the set A= (1, 2, 4, 5, 7,.8, 10, 14, 13, 14, 16, .....4 95, 97, 98, 100} of 67 elements in which there are no multiple of 3. As we have excluded 3, 6 we can include 9, 18) ‘As we have excluded 12, 15 can include 36, 45 ‘As we have excluded 21, 24 we can include 63, 72. AS we have excluded 30, 33, we can incltide 90, 99 ‘AS we have excluded 27, we can include 81 So, we can add each of the elements 18. 9, 18, 36, 45, 63, 72, 81, 90, 99 in set A at the most. So, the required set can have at most 67 + 9 = 76 members. Answer (21) O +x] 42 4 (1) aan} sada lec { Bs 64) 1 =Ax(16)9 +Bx, 43 Ld 28100 -2:0+5e4.10% (1) Ax(16)3 +Bx) 48 |4e Ud +e +e 4 5 20. Answer (02) ‘The set of these rational numbers is from 1 49 499 i to S where each element 2 has n+ m= ‘999° 501 m 1000 and © is irreducible. m fn _1000-m 1000 _, We note that is irreducible Hence, 2 is irreducible i 1000 and 1009 js irreducible if m is not divisible by 2 or 5. ‘There are 499 numbers between 501 and 999. 249 numbers are le by 2 ‘99 numbers are divisible by 5 49 numbers are divisible by 10 Using the principle of inclusion and exclusion, there are 499 — 249 ~ 99 + 49 = 200 numbers between 501 and 999 are not divisible by either 2 7. = 200 ck _200_ poy 706 "700 aa ALL RIGHTS RESERVED All rights including copyright and translation rights etc, reserved and vests exclusively with AESL. No part ofthis publication may be reproduced, distributed, redistributed, copied or transmitted in any form or by any means-graphical, electronic or mechanical methods including photocopying, recording, taping or stored on information retrieval systems of any nature or reproduced on any disc, tape, media, information storage device, without the prior written permission of AESL. Breach of this condition is liable for legal action (civil as well as criminal) under the applicable Laws, Edition: 2022-23 © Aakash Educational Services Limited [AESL]

You might also like